In Gandania

This topic has expert replies
Moderator
Posts: 7187
Joined: Thu Sep 07, 2017 4:43 pm
Followed by:23 members

In Gandania

by BTGmoderatorDC » Wed Sep 13, 2017 2:35 pm
In Gandania, where the government has a monopoly on tobacco sales, the incidence of smoking-related health problems has risen steadily for the last twenty years. The health secretary recently proposed a series of laws aimed at curtailing tobacco use in Gandania. Profits from tobacco sales, however, account for ten percent of Gandania's annual revenues. Therefore, Gandania cannot afford to institute the proposed laws.

Which of the following, if true, most seriously weakens the argument?

(A) All health care in Gandania is government-funded.

(B) Implementing the proposed laws is not likely to cause a significant increase in the amount of tobacco Gandania exports.

(C) The percentage of revenue Gandania receives from tobacco sales has remained steady in recent years.

(D) Profits from tobacco sales far surpass any other single source of revenue for the Gandanian government.

(E) No government official in Gandania has ever previously proposed laws aimed at curtailing tobacco use.

What is wrong with the other options? Can experts explain? Thanks

OA A

User avatar
Legendary Member
Posts: 2663
Joined: Wed Jan 14, 2015 8:25 am
Location: Boston, MA
Thanked: 1153 times
Followed by:128 members
GMAT Score:770

by DavidG@VeritasPrep » Thu Sep 14, 2017 6:42 am
lheiannie07 wrote:In Gandania, where the government has a monopoly on tobacco sales, the incidence of smoking-related health problems has risen steadily for the last twenty years. The health secretary recently proposed a series of laws aimed at curtailing tobacco use in Gandania. Profits from tobacco sales, however, account for ten percent of Gandania's annual revenues. Therefore, Gandania cannot afford to institute the proposed laws.

Which of the following, if true, most seriously weakens the argument?

(A) All health care in Gandania is government-funded.

(B) Implementing the proposed laws is not likely to cause a significant increase in the amount of tobacco Gandania exports.

(C) The percentage of revenue Gandania receives from tobacco sales has remained steady in recent years.

(D) Profits from tobacco sales far surpass any other single source of revenue for the Gandanian government.

(E) No government official in Gandania has ever previously proposed laws aimed at curtailing tobacco use.

What is wrong with the other options? Can experts explain? Thanks

OA A
Conclusion: Gandania cannot afford to implement anti-smoking regulation
Premises: Gandania gets 10% of its revenue from tobacco sales; smoking-related health problems have risen

If we're trying to weaken the conclusion, we want to show that Gandania can afford the regulation. Well, if the regulation is going to reduce revenue that the government would have received from tobacco sales, then the there must be some kind of ancillary benefit to the regulations, perhaps a reduction in expenses in another domain.

This is what A gives us. If smoking is causing health-related problems, and the government has to pay for all health care, then presumably, it's expensive for the government if a good deal of the population smokes. Thus, if the regulations reduce smoking rates, the money saved in health care expense may offset the profits forgone from lower tobacco sales.
Veritas Prep | GMAT Instructor

Veritas Prep Reviews
Save $100 off any live Veritas Prep GMAT Course